Đến nội dung

phudinhgioihan nội dung

Có 329 mục bởi phudinhgioihan (Tìm giới hạn từ 20-04-2020)



Sắp theo                Sắp xếp  

#542920 Nếu $|q|<1$ thì lim $q^n$ = 0

Đã gửi bởi phudinhgioihan on 04-02-2015 - 04:36 trong Dãy số - Giới hạn

Xin hỏi cách chứng minh định lý 2 trang 129 SGK đại số và giải tích 11 nâng cao
Địng lí 1 : Cho 2 dãy số $(u_n)$ và $(v_n)$ .
Nếu $|u_n|\leq v_n $ với mọi n và lim $v_n$ =0 thì lim $u_n$ =0 .

Áp dụng định lý 1 để chứng minh định lý 2 sau đây :
Nếu $|q|<1$ thì lim $q^n$=0

 

Áp dụng bài tập 7 trang 100:

Cho số thực $x>-1$, khi đó $(1+x)^n \ge 1+nx ,\; \forall n\in \mathbb{N}^*$

 

Vì $|q|<1$ nên $\frac{1}{|q|}>1$, do đó có số thực $p>0$ để $\frac{1}{|q|}=1+p \; \Leftrightarrow |q|=\frac{1}{1+p}$

 

$|q|^n=\frac{1}{(1+p)^n} \le \frac{1}{1+np} <\frac{1}{np} \;\;\;\forall n \in \mathbb{N}^*$

Do $\lim \frac{1}{np}=0$ nên $\lim |q|^n=0$ kéo theo $\lim q^n=0$




#542397 $\sum_{n=1}^{\infty}\frac{1}{n}(2^{\frac{1}{\sq...

Đã gửi bởi phudinhgioihan on 30-01-2015 - 22:37 trong Giải tích

xét sự hội tụ của:

$\sum_{n=1}^{\infty}\dfrac{1}{n}(2^{\frac{1}{\sqrt{n}}}-1)$

 

Ta có $e^x \underset{x \to 0}{\sim} 1+x$, do đó $2^{\frac{1}{\sqrt{n}}}=e^{\frac{\ln 2}{\sqrt{n}}} \underset{n \to +\infty}{\sim}1+\dfrac{\ln 2}{\sqrt{n}}$

 

Suy ra $\dfrac{2^{\frac{1}{\sqrt{n}}}-1}{n} \underset{n \to +\infty}{\sim} \dfrac{\ln 2}{n^\frac{3}{2}} $

 

$\forall n \ge 1, \; \dfrac{2^{\frac{1}{\sqrt{n}}}-1}{n} >0$ và $\sum_{n=1}^{+\infty} \dfrac{\ln 2}{n^\frac{3}{2}}$ hội tụ, do đó $\sum_{n=1}^{+\infty} \dfrac{2^{\frac{1}{\sqrt{n}}}-1}{n}$ cũng hội tụ.




#542223 Đề thi Olympic Toán sinh viên ĐH sư phạm HN 2015

Đã gửi bởi phudinhgioihan on 28-01-2015 - 23:43 trong Thảo luận về các kì thi, các kì kiểm tra Toán sinh viên

 


Bài 6:

Giả sử $\Re=\{ f\in C^2[0;1]:\; f(0)=f(1)=0, f^{'}(0)=a \}$

 

Tìm $\min_{f \in \Re} \int_0^1 \left( f^{''}(x) \right)^2 dx $

 

 

Trước tiên, ta thử tìm $f$ để $\int_0^1\left[ f^{''}(x) \right]^2dx=\min_{f \in \Re} \int_0^1 \left[ f^{''}(x) \right]^2 dx$, để đơn giản, thử tìm $f$ trong lớp hàm đa thức.

Nếu $f$ thoả mãn điều kiện trên, xét $g \in C^2[0;1]$ thoả mãn $g(0)=g(1)=g^{'}(0)=g^{'}(1)=0$ và $g \neq 0 \;\; (*)$, do đó $\int_0^1\left[ g^{''}(x) \right]^2dx>0$

 

$\forall t \in \mathbb{R}, \; f+tg \in \Re$ và $h(t)=\int_0^1\left[ f^{''}(x)+tg^{''}(x) \right]^2dx$ nhỏ nhất tại $t=0$, đồng thời

$h(t)=t^2\int_0^1\left[ g^{''}(x) \right]^2dx+2t\int_0^1 f^{''}(x)g^{''}(x)dx+\int_0^1\left[ f^{''}(x) \right]^2dx$ đạt giá trị nhỏ nhất tại $t=-\dfrac{\int_0^1 f^{''}(x)g^{''}(x)dx}{\int_0^1\left[ g^{''}(x) \right]^2dx}$ nên phải có $\int_0^1 f^{''}(x)g^{''}(x)dx=0$

 

Vậy, $0=\int_0^1 f^{''}(x)g^{''}(x)dx=\int_0^1 f^{'''}(x)g^{'}(x)dx=\int_0^1f^{(4)}(x)g(x) $ với vô số hàm $g$ thoả mãn $(*)$ nên suy ra $f^{(4)}(x)=0$ hay $f$ là đa thức bậc không quá 3.

Xét $f(x)=mx^3+nx^2+px+q$, kết hợp các điều kiện giả thiết và tìm $\min \int_0^1\left[ f^{''}(x) \right]^2dx$ ta được $f(x)=\dfrac{a}{2}x(x-1)(x-2)$

 

Giờ ta vào lời giải chính.

 

Cách 1:

 

Xét $f(x)=\frac{a}{2}x(x-1)(x-2)$. Dễ kiểm tra $f \in \Re$ và $\int_0^1\left[ f^{''}(x) \right]^2dx=3a^2$

 

Với $g \in \Re$ bất kỳ, $g=f+g-f=f+h \;\;, \left(h=g-f \right)$. Dễ có $h\in C^2[0;1],\; h(0)=h(1)=h^{'}(0)=0$

 

Ta có: $$\int_0^1 f^{''}(x)h^{''}(x)dx=3a\int_0^1 (x-1)h^{''}(x)dx=3a\int_0^1 g^{'}(x)dx=0$$

 

$$\int_0^1\left[ g^{''}(x) \right]^2dx=\int_0^1 \left( f^{''}(x)+h^{''}(x) \right)^2dx \\ =\int_0^1\left[ f^{''}(x) \right]^2dx+\int_0^1\left[ h^{''}(x) \right]^2dx=3a^2+\int_0^1\left[ h^{''}(x) \right]^2dx \\ \ge 3a^2 $$

 

Dấu $"="$ xảy ra khi và chỉ khi $h^{''}(x)=0 \Leftrightarrow g^{''}(x)=f^{''}(x) \;\;, \forall x \in [0;1]$

$$\Rightarrow \int_0^x g^{''}(t)dt=\int_0^x f^{''}(t)dt \;\;, \forall x \in [0;1]$$

$$\Leftrightarrow g^{'}(x)=f^{'}(x) \;\;, \forall x \in [0;1]$$

$$\Rightarrow \int_0^x g^{'}(t)dt=\int_0^x f^{'}(t)dt \;\;, \forall x \in [0;1]$$

$$\Leftrightarrow g(x)=f(x) \;\; \forall x \in [0;1]$$

 

Vậy $\min_{f \in \Re} \int_0^1 \left[ f^{''}(x) \right]^2 dx=3a^2$

 

Cách 2:

 

Từ lời giải ở trên và ý tưởng sử dụng bdt Cauchy-Schwarz dạng tích phân, ta có lời giải như sau:

Với $f \in \Re$ bất kỳ

$$\int_0^1 (x-1)^2dx \int_0^1\left[ f^{''}(x) \right]^2dx \ge \left( \int_0^1(x-1)f^{''}(x)dx \right)^2=\left(a-\int_0^1 f^{'}(x)dx \right)^2=a^2$$

 

$$\Leftrightarrow \frac{1}{3} \int_0^1\left[ f^{''}(x) \right]^2dx \ge a^2 $$

$$\Leftrightarrow \int_0^1\left[ f^{''}(x) \right]^2dx \ge 3a^2 $$

 

Dấu $"="$ xảy ra khi và chỉ khi $f^{''}(x)=k(x-1) \; \Rightarrow f(x)=\frac{a}{2}x(x-1)(x-2)$

 

Vậy $\min_{f \in \Re} \int_0^1 \left[ f^{''}(x) \right]^2 dx=3a^2$




#542122 Đề thi Olympic Toán sinh viên ĐH sư phạm HN 2015

Đã gửi bởi phudinhgioihan on 28-01-2015 - 00:22 trong Thảo luận về các kì thi, các kì kiểm tra Toán sinh viên

 

Bài 3:

Tìm tất cả các hàm liên tục $f: \mathbb{R} \to \mathbb{R}$ thỏa mãn

$$f(xf(y))=yf(x), \;\; \forall x,y \in \mathbb{R}$$

 

 

* Cho $x=1 \Rightarrow f(f(y))=yf(1) \;\; \forall y \in \mathbb{R}$

 $ \Rightarrow f(f(xf(y))=xf(y)f(1)\;\; \forall x,y \in \mathbb{R}$

Nhưng $f(f(xf(y))=f(yf(x))=xf(y)$ suy ra $xf(y)=xf(y)f(1) \;\; \forall x,y \in \mathbb{R}$

 

$$\Rightarrow f(x)=0\;\;\forall x \in \mathbb{R} \; \vee \; f(1)=1$$

 

+ $f(x)=0\;\; \forall x \in \mathbb{R}$ thay vào thấy thỏa giả thiết.

 

+$f(1)=1$  $\Rightarrow f(f(x))=x \;\; \forall x \in \mathbb{R}$

Suy ra $f\circ f$ là song ánh do đó $f$ là song ánh $\mathbb{R} \to \mathbb{R}$

 

Từ giả thiết, cho $x=0$, $f(0)=yf(0) \;(\forall y \in \mathbb{R}) \; \Rightarrow f(0)=0$

Với $y \in \mathbb{R}, \exists ! t \in \mathbb{R},\; y=f(t) \;\Rightarrow f(xt)=f(xf(y))=yf(x)=f(t)f(x)$

 

Suy ra $f(xy)=f(x)f(y) \;\;, \forall x,y \in \mathbb{R}$

$\Rightarrow f(x^n)=\left(f(x) \right)^n \;\; \forall n \in \mathbb{N}^*$

 

* Nếu $|x|<1 \; \Rightarrow \lim_{n \to +\infty} \left(f(x) \right)^n=\lim_{n \to +\infty} f(x^n)=f(0)=0 \; \Rightarrow |f(x)|<1$

* Nếu $|x|>1 \; \Rightarrow \lim_{n \to +\infty} \left(f(x) \right)^n=\lim_{n \to +\infty} f(x^n)=\infty \; \Rightarrow |f(x)|>1$

 

Đặt $A=(-1;1),\; B=(-\infty;-1) \cup (1;+\infty)$, suy ra $f(A) \subset A,\; f(B) \subset B$

 

Với $x \neq 0$,  $ f(x)f(\frac{1}{x})=f(1)=1 \; \Rightarrow f(\frac{1}{x})=\dfrac{1}{f(x)}$

Nếu có $x_0 \in \mathbb{R}$ sao cho $f(x_0) \neq x_0 $ (dễ có $x_0 \neq 0$ và $f(x_0)\neq 0$ do $f$ là song ánh),

$$f(\frac{x_0}{f(x_0)})=f(x_0f(\frac{1}{x_0}))=\frac{f(x_0)}{x_0}$$

 

Nhưng $\frac{x_0}{f(x_0)}$ và $\frac{f(x_0)}{x_0}$ không cùng thuộc $A$ và cũng không cùng thuộc $B$ nên $f(\frac{x_0}{f(x_0)} )\neq \frac{f(x_0)}{x_0}$, mâu thuẫn.

Vậy $f(x)=x \; \forall x \in \mathbb{R}$

 

Thử lại, hàm $f(x)=x$ thỏa mãn giả thiết.

 

Kết luận: có 2 hàm số thỏa mãn là: $f(x)=0$ hoặc $f(x)=x$




#542090 $$u^2+v^2+w^2 \ge \dfrac{2(abc)^2}{a^2b^2+...

Đã gửi bởi phudinhgioihan on 27-01-2015 - 21:21 trong Hình học không gian

Bài toán :
Cho tứ diện $S.ABC$ có $SA=a, SB=b, SC=c$ đôi một vuông góc với nhau. Lấy điểm $M$ nằm trong tam giác $ABC$ Gọi $u, v, w$ lầ lượt là khoảng cách từ $M$ đến $SA, SB, SC$. Chứng minh rằng :
$$u^2+v^2+w^2 \ge \dfrac{2(abc)^2}{a^2b^2+b^2c^2+c^2a^2}$$.

 

Gọi $h_a,h_b,h_c$ lần lượt là khoảng cách từ $M$ đến $(SBC),(SCA),(SAB)$.

 

Ta có: $\dfrac{h_a}{a}=\dfrac{h_a.S_{SBC}}{a.S_{SBC}}=\dfrac{V_{MSBC}}{SABC}$

Tương tự, $\dfrac{h_b}{b}=\dfrac{V_{MSAC}}{V_{SABC}},\; \dfrac{h_c}{c}=\dfrac{V_{MSAB}}{V_{SABC}}$

 

Suy ra $$\dfrac{h_a}{a}+\dfrac{h_b}{b}+\dfrac{h_c}{c}=\dfrac{V_{MSBC}+V_{MSAC}+V_{MSAB}}{v_{SABC}}=1$$

 

Theo bdt $Cauchy-Schwarz$, $$1 \le (h_a^2+h_b^2+h_c^2)(\dfrac{1}{a^2}+\frac{1}{b^2}+\frac{1}{c^2})$$

$$\Leftrightarrow h_a^2+h_b^2+h_c^2 \ge \dfrac{(abc)^2}{a^2b^2+b^2c^2+c^2a^2}$$

 

Gọi $M_a$ là hình chiếu vuông góc của $M$ trên $(SBC)$, $M_{ab},M_{ac}$ lần lượt là hình chiếu vuông góc của $M_a$ trên $SB,SC$

 

Ta có: $u^2=SM_a^2=SM_{ab}^2+M_{a}M_{ab}^2=h_b^2+h_c^2$

Tương tự, $v^2=h_a^2+h_c^2,\; w^2=h_a^2+h_b^2$

 

Suy ra $$u^2+v^2+w^2=2(h_a^2+h_b^2+h_c^2)\ge \dfrac{2(abc)^2}{a^2b^2+b^2c^2+c^2a^2} \;\;\;(*)$$

 

Gọi $H$ là trực tâm của $\Delta ABC$ (và cũng là hình chiếu vuông góc của $S$ trên $(ABC)$)

 

Ta có: $\dfrac{1}{SH^2}=\dfrac{1}{a^2}+\frac{1}{b^2}+\frac{1}{c^2}$ và $SM^2=h_a^2+u^2=h_a^2+h_b^2+h_c^2$

Do đó, đẳng thức trong bất đẳng thức $(*)$ xảy ra khi và chỉ khi $M \equiv H$




#542029 Đề thi Olympic Toán sinh viên ĐH sư phạm HN 2015

Đã gửi bởi phudinhgioihan on 27-01-2015 - 03:57 trong Thảo luận về các kì thi, các kì kiểm tra Toán sinh viên


Bài 4:

Giả sử $f:[a;b] \to \mathbb{R}\;,\; b-a\ge 4$ là hảm khả vi trên $(a;b)$. Chứng minh tồn tại $x_0\in (a;b)$ sao cho

$$f^{'}(x_0)<1+f^2(x_0)$$

 

 

Theo định lý Lagrange, $$\exists \;x_0 \in (a;b),\; \arctan f(b)-\arctan f(a)=(b-a)\dfrac{f^{'}(x_0)}{1+f^2(x_0)} \\ \Rightarrow \dfrac{f^{'}(x_0)}{1+f^2(x_0)}=\dfrac{\arctan f(b)-\arctan f(a)}{b-a} \le \dfrac{\pi}{b-a} \le \dfrac{\pi}{4}<1 \\ \Rightarrow f^{'}(x_0)<1+f^2(x_0)$$




#542027 Đề thi Olympic Toán sinh viên ĐH sư phạm HN 2015

Đã gửi bởi phudinhgioihan on 27-01-2015 - 03:50 trong Thảo luận về các kì thi, các kì kiểm tra Toán sinh viên

 

 

Bài 5:

Giả sử $f:[-1;1] \to \mathbb{R}$ là hàm khả vi đến cấp 3 và biết rằng $f(-1)=f(0)=f^{'}(0)=0,f(1)=1$. Chứng minh rằng tồn tại $c \in (-1;1)$ sao cho $f^{'''}( c ) \ge 3 $

 

 

Khai triển Taylor của $f(x)$ tại $0$: $f(x)=\frac{x^2}{2}f^{''}(0)+\frac{x^3}{6}f^{'''}(c_x) \;\;, \; c_x \in (-1;1)$

 

Như vậy, tồn tại $c_1,c_2 \in (-1;1)$ sao cho $f(-1)=\frac{1}{2}f^{''}(0)-\frac{1}{6}f^{'''}(c_1),\;\;f(1)=\frac{1}{2}f^{''}(0)+\frac{1}{6}f^{'''}(c_2)$

 

$\Rightarrow f^{'''}(c_1)+f^{'''}(c_2)=6(f(1)-f(-1))=6$

 

Đặt $c$ sao cho $f^{'''}(c )=\max \{ f^{'''}(c_1);f^{'''}(c_2)\}$, khi đó $6=f^{'''}(c_1)+f^{'''}(c_2) \le 2f^{'''}(c )$

 

$\Rightarrow f^{'''}(c ) \ge 3$




#542026 Đề thi Olympic Toán sinh viên ĐH sư phạm HN 2015

Đã gửi bởi phudinhgioihan on 27-01-2015 - 03:38 trong Thảo luận về các kì thi, các kì kiểm tra Toán sinh viên

 

Đề GIẢI TÍCH

 


 

Bài 2:

Cho $f:[0;1] \to \mathbb{R}$ là hàm thỏa mãn $f(0)<0,\; f(1)>0$ . Giả sử tồn tại hàm liên tục $g$ trên $[0;1]$ sao cho hàm $f+g$ giảm trên $[0;1]$. Chứng minh rằng phương trình $f(x)=0$ có nghiệm trong $(0;1)$.

 

 

Gọi $A=\{x \in [0;1], f(x) \le 0\}$. Do $0 \in A$ và $A$ bị chặn nên $\exists \; a =\sup (A)$

Theo định nghĩa của $A$ suy ra $f(a) \le 0$.

 

Do $f+g$ đơn điệu giảm nên $g(a) \ge f(a)+g(a) \ge f(1)+g(1)>g(1)$

Do $g$ liên tục trên $[a;1]$ nên $\exists t \in [a;1),\; g(t)=f(a)+g(a)$

 

Lại có $f(t)+g(t) \le f(a)+g(a) \Rightarrow f(t) \le 0$ nên $t \in A$, suy ra $t \le a$, do đó $t=a$

Suy ra $g(a)=g(t)=f(a)+g(a) \Leftrightarrow f(a)=0$

 

Vậy $f(x)=0$ có nghiệm trong $(0;1)$




#541948 Đề thi Olympic Toán sinh viên ĐH sư phạm HN 2015

Đã gửi bởi phudinhgioihan on 26-01-2015 - 18:06 trong Thảo luận về các kì thi, các kì kiểm tra Toán sinh viên

 

Bài 5:

Cho bất phương trình

$$\frac{1}{x-1}+\frac{1}{x-2}+\frac{1}{x-3}+\frac{1}{x-4}>2015$$

 

Hãy tìm tổng độ dài các khoảng nghiệm trên trục số.

 

 

Bài 2,4 lười gõ quá, bài 6 thì quá quen biết.

 

$$\frac{1}{x-1}+\frac{1}{x-2}+\frac{1}{x-3}+\frac{1}{x-4}>2015$$

$\Leftrightarrow \dfrac{(x-1)(x-2)(x-3)+(x-1)(x-2)(x-4)+(x-2)(x-3)(x-4)+(x-1)(x-3)(x-4)-2015(x-1)(x-2)(x-3)(x-4)}{(x-1)(x-2)(x-3)(x-4)}>0$

 

Đặt

$f(x)=(x-1)(x-2)(x-3)+(x-1)(x-2)(x-4)+(x-2)(x-3)(x-4)+(x-1)(x-3)(x-4)-2015(x-1)(x-2)(x-3)(x-4)$

 

$g(x)=(x-1)(x-2)(x-3)(x-4)$

 

$f(x)$ là đa thức bậc 4 với hệ số bậc cao nhất là $-2015$, hệ số bậc 3 là $20154$

 

Ta có:

 $$\lim_{x \to +\infty}f(x)=-\infty \;, \lim_{x \to -\infty} f(x)=-\infty \\ f(1)<0 ,\; f(2)>0,\; f(3)<0,\; f(4)>0$$

 

Suy ra $f$ có 4 không điểm thực $x_1,x_2,x_3,x_4$ với $x_1 \in (1;2),\; x_2\in (2,3),\; x_3\in (3,4),\; x_4\in (4;+\infty)$

 

Theo định lý Viète, $x_1+x_2+x_3+x_4=\frac{20154}{2015}$

 

Lập bảng xét dấu, ta được tập nghiệm $S$ của bất phương trình đã cho là

 

$$S=\{ (1;x_1), \; (2;x_2),\;(3;x_3),\;(4,x_4) \}$$

 

Vậy tổng độ dài các khoảng nghiệm bằng $x_1+x_2+x_3+x_4-10=\frac{4}{2015}$




#541944 Đề thi Olympic Toán sinh viên ĐH sư phạm HN 2015

Đã gửi bởi phudinhgioihan on 26-01-2015 - 17:41 trong Thảo luận về các kì thi, các kì kiểm tra Toán sinh viên


 

Bài 3:

Cho $A$ là ma trận thực vuông cấp $n \in \mathbb{N}^*$ thỏa mãn $A^3=A+I_n$ với $I_n$ là ma trận đơn vị cấp $n$. Chứng minh rằng $\det A>0$

 

 

 

Ta có $ A^3=A+I_n \Leftrightarrow A(A^2-I)=I_n$ do đó $A$ khả nghịch nên $|A| \neq 0$

 

$$A(A+I)(A-I)=I_n \Leftrightarrow A^4(A-I)=I_n \Rightarrow |A|^4|A-I_n|=1 \Rightarrow |A-I_n|>0$$

 

$$A^3=A+I_n \Leftrightarrow (A-I_n)(A^2+A+I_n)=A \\  \Rightarrow |A|=|A-I_n||A^2+A+I_n| \; \wedge \; |A^2+A+I_n| \neq 0 \;\; (1 )$$

 

$$\left|A^2+A+I_n\right|= \left|(A+\frac{1}{2}I_n )^2+( \frac{\sqrt{3}}{2} I_n )^2\right| \\=\left|A+\frac{1}{2}I_n+\frac{\sqrt{3}}{2}i I_n\right|\left|A+\frac{1}{2}I_n-\frac{\sqrt{3}}{2}i I_n\right| \\ =\left|A+\frac{1}{2}I_n+\frac{\sqrt{3}}{2}i I_n\right| \overline{\left|A+\frac{1}{2}I_n+\frac{\sqrt{3}}{2}i I_n\right|} \\ ={\left|A+\frac{1}{2}I_n+\frac{\sqrt{3}}{2}i I_n\right|}^2 > 0 \;\;(2 )$$

 

 

Từ $( 1)$ và $( 2 )$ suy ra $|A|>0$




#541938 Đề thi Olympic Toán sinh viên ĐH sư phạm HN 2015

Đã gửi bởi phudinhgioihan on 26-01-2015 - 16:53 trong Thảo luận về các kì thi, các kì kiểm tra Toán sinh viên

 

Đề ĐẠI SỐ

 

Bài 1:

Cho $A,B$ là hai ma trận vuông thực cấp $n \in \mathbb{N}^*$. Giả sử tồn tại $n+1$ số thực phân biệt $t_1,...,t_{n+1}$ sao cho

$C_i=A+t_iB,\;i=1,...,n+1$ là lũy linh. Hãy chứng minh rằng $A,B$ là các ma trận lũy linh.


 

 

 Khai triển $(A+tB)^n$ được:

$$(A+tB)^n=A^n+\sum_{i=1}^{n-1}t^iP_i(A,B)+t^nB^n \;\; ( * )$$

Trong đó $P_i(A,B),\;\left( i\in\{1,...,n-1\} \right)$ là đa thức ma trận theo $A$ và $B$.

 

Giả sử $(A+tB)^n=(c_{ij})_{1 \le i,j \le n}$

 

Xét một phần tử $c_{ij}$ bất kỳ của $(A+tB)^n$, theo $( * )$ thì $c_{ij}$ là một đa thức hệ số thực biến $t$ bậc không vượt quá $n$, ký hiệu là $Q_{ij}(t)$

 

Do $C_i$ lũy linh $\forall i\in \{1,...,n+1\}$, do đó $(A+t_iB)^n=C_i^n=0 ,\; \forall i \in \{1,...,n+1\}$

Suy ra $Q_{ij}(t)$ có $n+1$ nghiệm thực phân biệt là $t_1,...,t_{n+1}$, do đó $Q_{ij}(t)=0 ,\; \forall t \in \mathbb{R}$

 

Hệ số tự do và hệ số của $t^n$ trong $Q_{ij}(t)$ lần lượt là $(A^n)_{ij}$ và $(B^n)_{ij}$, do đó $(A^n)_{ij}=(B^n)_{ij}=0$

Như vậy, mọi phần tử của $A^n$ và $B^n$ đều bằng 0 hay $A^n=B^n=0$, do vậy $A$ và $B$ cùng lũy linh.




#541934 Đề thi Olympic Toán sinh viên ĐH sư phạm HN 2015

Đã gửi bởi phudinhgioihan on 26-01-2015 - 15:22 trong Thảo luận về các kì thi, các kì kiểm tra Toán sinh viên

Đề ĐẠI SỐ

 

Bài 1:

Cho $A,B$ là hai ma trận vuông thực cấp $n \in \mathbb{N}^*$. Giả sử tồn tại $n+1$ số thực phân biệt $t_1,...,t_{n+1}$ sao cho

$C_i=A+t_iB,\;i=1,...,n+1$ là lũy linh. Hãy chứng minh rằng $A,B$ là các ma trận lũy linh.

 

Bài 2:

Cho $a_0$ và $d$ là các số thực.Với $j=0,...,n$, đặt $a_j=a_0+jd$. Cho

$$A=\begin{pmatrix}a_0 &a_1 &a_2 &\cdots &a_n \\a_1 &a_0 &a_2 &\cdots & a_{n-1}\\ a_2 &a_1 &a_0 &\cdots &a_{n-2} \\ \vdots &\vdots &\vdots &\vdots &\vdots \\a_n &a_{n-1} &a_{n-2} &\cdots &a_0 \end{pmatrix}$$

 

Hãy tính định thức của $A$ theo $a_0,d,n$

 

Bài 3:

Cho $A$ là ma trận thực vuông cấp $n \in \mathbb{N}^*$ thỏa mãn $A^3=A+I_n$ với $I_n$ là ma trận đơn vị cấp $n$. Chứng minh rằng $\det A>0$

 

Bài 4:

Tìm tất cả ma trận thực vuông $A$ cấp $2$ sao cho $A^2=I_2$ với $I_2$ là ma trận đơn vị cấp 2.

 

Bài 5:

 

Cho bất phương trình

$$\frac{1}{x-1}+\frac{1}{x-2}+\frac{1}{x-3}+\frac{1}{x-4}>2015$$

 

Hãy tìm tổng độ dài các khoảng nghiệm trên trục số.

 

Bài 6:

Cho $P(x)$ là đa thức bậc $n\in \mathbb{N}^*$ với các hệ số thực và chỉ có nghiệm thực. Chứng minh rằng

 

$$(n-1)\left(P^{'}(x) \right)^2 \ge nP(x)P^{''}(x)$$

 

 

Đề GIẢI TÍCH

 

Bài 1:

Cho hai số thực dương $a$ và $a_1$. Định nghĩa dãy $(a_n)_{n \ge 1}$ bới

$$a_{n+1}=a_n(2-aa_n) ,\; \forall n \in \mathbb{N}^*$$

 

Xét sự hội tụ và tìm giới hạn (nếu có) của $(a_n)_{n \ge 1}$

 

Bài 2:

Cho $f:[0;1] \to \mathbb{R}$ là hàm thỏa mãn $f(0)<0,\; f(1)>0$ . Giả sử tồn tại hàm liên tục $g$ trên $[0;1]$ sao cho hàm $f+g$ giảm trên $[0;1]$. Chứng minh rằng phương trình $f(x)=0$ có nghiệm trong $(0;1)$.

 

Bài 3:

Tìm tất cả các hàm liên tục $f: \mathbb{R} \to \mathbb{R}$ thỏa mãn

$$f(xf(y))=yf(x), \;\; \forall x,y \in \mathbb{R}$$

 

Bài 4:

Giả sử $f:[a;b] \to \mathbb{R}\;,\; b-a\ge 4$ là hảm khả vi trên $(a;b)$. Chứng minh tồn tại $x_0\in (a;b)$ sao cho

$$f^{'}(x_0)<1+f^2(x_0)$$

 

Bài 5:

Giả sử $f:[-1;1] \to \mathbb{R}$ là hàm khả vi đến cấp 3 và biết rằng $f(-1)=f(0)=f^{'}(0)=0,f(1)=1$. Chứng minh rằng tồn tại $c \in (-1;1)$ sao cho $f^{'''}( c ) \ge 3 $

 

Bài 6:

Giả sử $\Re=\{ f\in C^2[0;1]:\; f(0)=f(1)=0, f^{'}(0)=a \}$

 

Tìm $\min_{f \in \Re} \int_0^1 \left( f^{''}(x) \right)^2 dx $




#541932 Tính khả vi và tính liên tục

Đã gửi bởi phudinhgioihan on 26-01-2015 - 14:23 trong Giải tích

Cho em hỏi là tại sao các bài toán hay nói là liên tục trên $[a,b]$ và khả vi trên $(a,b)$ mà không phải là khả vi trên $[a,b]$.

 

 

Và hình như là nếu khả vi thì liên tục vậy tại sao ko bỏ luôn cái câu liên tục trên $[a,b]$ ạ?

 

Giải thích một cách sơ cấp, định nghĩa đạo hàm tại một điểm $f^{'}(x)=\lim_{h \to 0} \dfrac{f(x+h)-f(x)}{h}$, như vậy $f^{'}(x)$ tồn tại khi và chỉ khi $\lim_{h \to 0^+} \dfrac{f(x+h)-f(x)}{h}=\lim_{h \to 0^-} \dfrac{f(x+h)-f(x)}{h}$

 

Như vậy, nếu $x=a$ hoặc $x=b$ thì ta chỉ có $\lim_{h \to 0^+} \dfrac{f(a+h)-f(a)}{h},\;\lim_{h \to 0^-} \dfrac{f(b+h)-f(b)}{h}$ nên không thể nói được gì về tính khả vi tại hai điểm này.




#541401 Đề thi Olympic Toán sinh viên ĐH Kinh tế quốc dân HN 2015

Đã gửi bởi phudinhgioihan on 20-01-2015 - 11:46 trong Thảo luận về các kì thi, các kì kiểm tra Toán sinh viên

Bài 3 chắc là không phải dấu = :'( .

 

 

Anh đã sửa, là dấu $\le$ :D




#541381 Đề thi Olympic Toán sinh viên ĐH Kinh tế quốc dân HN 2015

Đã gửi bởi phudinhgioihan on 19-01-2015 - 23:09 trong Thảo luận về các kì thi, các kì kiểm tra Toán sinh viên

Môn GIẢI TÍCH

Bài 1:

Cho dãy $\{x_n\}$ được xác định như sau

$$x_1>0, \;\;\;, x_{n+1}=\dfrac{x_n^3+4x_n}{x_n^2+1},\; n \ge 1$$

 

Chứng minh rằng $\lim_{n \to +\infty} \dfrac{x_n}{n^2}=0$

 

Bài 2:

Cho hàm số $f(x)$ thỏa mãn các điều kiện: $f(x)>0$, đơn điệu tăng và $f^{''}(x)<0 \;, \forall x>0$. Chứng minh tồn tại số $M$ sao cho

$$f(x)<Me^\frac{x}{2014} ,\; \forall x>0$$

 

Bài 3:

Cho $f:[0;1] \to [0;+\infty)$ khả vi và hàm đạo hàm cấp một đơn điệu giảm và $f(0)=0,\; f^{'}(1)>0$, chứng minh

$$\int_{0}^1 \dfrac{dx}{f^2(x)+1} \le \dfrac{f(1)}{f^{'}(1)}$$

Khi nào đẳng thức xảy ra?

 

Bài 4:

Cho $f,g: \mathbb{R} \to [0;+\infty)$ là các hàm số liên tục thỏa mãn

$$\left| f(x)-x \right| \le g(x)-g(f(x)) \;, \forall x \in \mathbb{R}$$

Chứng minh rằng phương trình $f(x)=x$ có nghiệm.

 

Bài 5:

Chứng minh nếu $f(x)$ là hàm số liên tục và $f(x) \neq x\;, \forall x \in \mathbb{R}$ thì $f(f(x)) \neq x ,\; \forall x \in \mathbb{R}$

 

Bài 6:

Tìm hàm số $f(x)$ đơn điệu tăng trên $\mathbb{R}$ thỏa mãn:

  • $f(-x)=-f(x), \; \forall x \in \mathbb{R}$

  • $f(x+1)=f(x)+1 ,\; \forall x \in \mathbb{R}$

  • $f(\frac{1}{x})=\dfrac{f(x)}{x^2},\; \forall x \neq 0$

 

 

Môn ĐẠI SỐ

 

Bài 1:

Cho các số thực $a_1,...,a_n,x_1,...,x_n , \; n \ge 2$, tính định thức cấp $n$ sau:

 

$$D=\begin{vmatrix}a_1& a_2& a_3& \cdots& a_{n-1}&a_n\\-x_1 &x_2 &0 &\cdots &0 &0  \\0 &-x_2 &x_3 &\cdots &0 &0  \\\cdots \cdots&\cdots &\cdots &\cdots &\cdots &\cdots  \\0 &0 & 0& \cdots& -x_{n-1}& x_n\end{vmatrix}$$

 

Bài 2:

Cho $A,B$ là các ma trận thực cấp $n$ khác nhau thỏa mãn $A^3=B^3,\; A^2B=B^2A$. Chứng minh $A^2+B^2$ suy biến.

 

Bài 3:

Cho $A,B$ là các ma trận vuông khả nghịch, chứng minh nếu $A+B$ khả nghịch thì $A^{-1}+B^{-1}$ cũng khả nghịch.

 

Bài 4:

Cho $A$ là ma trận thức cấp $4\times 2$ và $B$ là ma trận thực cấp $2 \times 4$ sao cho

$$AB=\begin{pmatrix}1&0&-1&0 \\0&1&0&-1 \\-1&0&1&0 \\0&-1&0&1 \end{pmatrix}$$

 

Tìm $BA$

 

Bài 5:

 

Xác định ma trận $A$ biết rằng ma trận phụ hợp của nó là

$$A^{*}=\begin{pmatrix} m^2-1&1-m&1-m\\1-m&m^2-1&1-m \\1-m&1-m&m^2-1 \end{pmatrix}\;, m \neq 1, m\neq -2$$

 

Bài 6:

 

Cho hệ phương trình

 

$$\begin{cases} a_{11}x_1+a_{12}x_2+a_{13}x_3=0\\a_{21}x_1+a_{22}x_2+a_{23}x_3=0 \\ a_{31}x_1+a_{32}x_2+a_{33}x_3=0\end{cases}$$

 

Với các hệ số $a_{ij}$ thỏa mãn

  • $a_{11},a_{22},a_{33}$ là các số thực dương
  • Tất cả các hệ số khác âm
  • Trong mỗi phương trình, tổng các hệ số là số thực dương

Chứng minh hệ phương trình đã cho có nghiệm duy nhất $x_1=x_2=x_3=0$




#541269 $\begin{pmatrix} A & B\\ C & D \en...

Đã gửi bởi phudinhgioihan on 19-01-2015 - 00:03 trong Đại số tuyến tính, Hình học giải tích

Trường hợp $rank A=n$

Các hàng của ma trận $\left ( C,D \right )$ là tổ hợp tuyến tính của các hàng của $\left ( A,B \right )$ nên $\left ( C,D \right )=Q\left ( A,B \right )\Rightarrow C=QA,D=QB\Rightarrow \left | C \right |=\left | Q \right |\left | A \right |,\left | D \right |=\left | Q \right |\left | B \right |$. Suy ra điều phải chứng minh (hai hàng tỷ lệ với nhau).

(hình như hai cách giải bản chất như nhau).

 

Trường hợp $rank A < n$ (như anh phudinhgioihan)

 

Nếu mà giải chi tiết ra thì vẫn còn khá dài, xin chốt lời giải ngắn nhất vậy (có lẽ :D)

 

Do $rank \begin{pmatrix} A& B\\ C&D \end{pmatrix}=n$ nên bằng các phép biến đổi sơ cấp, ta đưa được $\begin{pmatrix} A& B\\C &D \end{pmatrix}$ về dạng $\begin{pmatrix} I_n& O\\O &O \end{pmatrix}$,

do đó, tồn tại $P,Q \in \mathbf{M}_{2n}(\mathbb{C})$ khả nghịch sao cho $\begin{pmatrix}A &B \\C &D \end{pmatrix}=P \begin{pmatrix} I_n& O\\O &O \end{pmatrix}Q$

 

Phân tích $P=\begin{pmatrix}P_1 &P_2 \\ P_3& P_4\end{pmatrix} \;\;, Q=\begin{pmatrix}Q_1 &Q_2 \\Q_3 & Q_4\end{pmatrix},\;\; P_i,Q_i \in \mathbf{M}_n(\mathbb{C}) \; \forall \; i \in \{1;2;3;4\}$

 

Suy ra $$\begin{pmatrix} A& B\\ C&D \end{pmatrix}=\begin{pmatrix}P_1 &P_2 \\ P_3& P_4\end{pmatrix}\begin{pmatrix} I_n& O\\O &O \end{pmatrix}\begin{pmatrix}Q_1 &Q_2 \\Q_3 & Q_4\end{pmatrix}=\begin{pmatrix}P_1Q_1 &P_1Q_2 \\P_3Q_1 & P_3Q_2\end{pmatrix}$$

 

$$\Rightarrow A=P_1Q_1\;\;,B=P_1Q_2\;\;, C=P_3Q_1\;\;, D=P_3Q_2$$

 

$$\Rightarrow |A||D|=|P_1||P_3||Q_1||Q_2|=|B||C|$$

$$\Leftrightarrow \begin{vmatrix}|A| &|B| \\|C| & |D|\end{vmatrix}=0$$

 

 

Nói thêm, nếu $A$ khả nghịch, do đó $P_1,Q_1$ cũng khả nghịch, suy ra $D=P_3Q_1{Q_1}^{-1}{P_1}^{-1}P_1Q_2=CA^{-1}B$




#541199 Kỷ yếu Olympic Toán sinh viên 2014

Đã gửi bởi phudinhgioihan on 18-01-2015 - 18:04 trong Thảo luận về các kì thi, các kì kiểm tra Toán sinh viên

Kỷ yếu gồm có: giới thiệu về trường ĐH Phạm Văn Đồng (Quãng Ngãi)-nơi tổ chức kỳ thi năm 2014, đề thi chính thức 2 môn ĐS và GT, một số bài dự tuyển của các trường.

File gửi kèm




#541197 Thông báo kỳ thi Olympic Toán sinh viên 2015

Đã gửi bởi phudinhgioihan on 18-01-2015 - 17:24 trong Thảo luận về các kì thi, các kì kiểm tra Toán sinh viên

ĐỊA ĐIỂM:
Trường đại học Kinh tế - Đại học Huế
Số 3 Lê Lợi, thành phố Huế, Thừa Thiên Huế
 
THỜI GIAN: 13-19/4/2015
 
BAN TỔ CHỨC:

  • Các đồng trưởng ban: GS. TSKH. Phùng Hồ Hải - Tổng thư ký Hội Toán học Việt Nam và PGS. TS. Trần Văn Hòa - Hiệu trưởng Trường đại học Kinh tế, Đại học Huế.
  • Phó ban: GS. TSKH. Phạm Thế Long - Phó chủ tịch Hội THVN; PGS. TS. Nguyễn Tài Phúc - Phó hiệu trưởng Trường đại học Kinh tế - Đại học Huế; GS. TS. Lê Văn Thuyết - UV BCH Hội THVN và ĐH Huế; Đại diện Bộ GD và ĐT; Đại diện TW Hội Sinh viên Việt Nam.
  • Uỷ viên: TS. Đoàn Trung Cường - Viện Toán học; TS. Lê Cường - ĐH Bách khoa Hà Nội; ThS. Ngô Sỹ Hùng - Trường đại học Kinh tế - Đại học Huế.

NỘI DUNG THI:
 

Môn ĐẠI SỐ

 

Screenshot223_zps6b33c517.png

Screenshot224_zps444c8f23.png

 

 

Môn GIẢI TÍCH

 

Screenshot225_zpse7852e20.png

Screenshot226_zps14bd009c.png


 




#541093 $\sum_{n=3}^{+\infty }\frac{...

Đã gửi bởi phudinhgioihan on 17-01-2015 - 15:11 trong Giải tích

Xét tính hội tụ của $\sum_{n=3}^{+\infty }\frac{\left| {\sin n} \right|+\left| {\cos n} \right|}{\sqrt{n^3-3n+3}}$

 

$\forall n \ge 3,\; 0<\dfrac{|\sin n|+|\cos n|}{\sqrt{n^3-3n+3}} \le \dfrac{\sqrt{2(\sin^2n+\cos^2n)}}{\sqrt{n^3-3n+3}} < \dfrac{\sqrt{2}}{\sqrt{\frac{n^3}{2}}}=\dfrac{2}{n^\frac{3}{2}}$

 

$\sum_{n=3}^{+\infty} \frac{2}{n^\frac{3}{2}} $ hội tụ nên $\sum_{n=3}^{+\infty }\frac{\left| {\sin n} \right|+\left| {\cos n} \right|}{\sqrt{n^3-3n+3}}$ hội tụ.




#541025 $\begin{pmatrix} A & B\\ C & D \en...

Đã gửi bởi phudinhgioihan on 16-01-2015 - 17:52 trong Đại số tuyến tính, Hình học giải tích

Câu 2. Cho $A,B,C,D\in Mat(n\times n,\mathbb{C})$. Chứng minh rằng nếu ma trận $\begin{pmatrix} A & B\\ C & D \end{pmatrix}$ có hạng bằng $n$ thì $$\begin{vmatrix} \left |A \right | & \left |B \right |\\ \left |C \right | & \left |D \right | \end{vmatrix}=0$$ 

 

Nếu $|A| \neq 0$, tức $A^{-1}$ tồn tại thì

 

$$\begin{pmatrix}I &O \\-CA^{-1} &I\end{pmatrix} \begin{pmatrix}A &B \\C &D \end{pmatrix}=\begin{pmatrix}A &B \\0 &D-CA^{-1}B \end{pmatrix}$$

 

$$\Rightarrow rank \begin{pmatrix}A &B \\0 &D-CA^{-1}B \end{pmatrix}=rank \begin{pmatrix}A &B \\C &D \end{pmatrix}=n$$

 

Ta chứng minh bất đẳng thức:

$$rank \begin{pmatrix}M & T\\ O& N\end{pmatrix} \ge rank M+rank N \;\;, (M,N,T) \in \left(\mathbf{M}_{n}(\mathbb{C})\right)^3$$

 

Giả sử, $rank M=p, \; rank N=q $, khi đó $M$ có p cột độc lập tuyến tính là các $m_i , \;( i=1,...,p)$ và $N$ có $q$ cột độc lập tuyến tính là các $n_i, \; (i=1,...,q)$.

Ký hiệu $t_i, \; (i=1,...,q)$ là các cột tương ứng của $T$ nằm trên $n_i$ trong $\begin{pmatrix}M & T\\O & N\end{pmatrix}$

 

Xét tổ hợp tuyến tính $x_1\begin{pmatrix} m_1 \\ 0 \end{pmatrix}+...+x_p\begin{pmatrix} m_p \\ 0 \end{pmatrix}+y_1\begin{pmatrix} t_1 \\ n_1 \end{pmatrix}+...+y_q\begin{pmatrix} t_q \\ n_q \end{pmatrix}=0$

 

$$\Rightarrow \sum_{i=1}^p x_i m_i+\sum_{i=1}^q y_i t_i=0 \; \wedge\; \sum_{i=1}^q y_in_i=0$$

 

$$\Rightarrow \sum_{i=1}^p x_im_i=0 \; \wedge \; y_j=0 ,\; (j=1,...,q)$$

 

$$\Rightarrow x_i=0, (\;i=1,...,p) \;\;, y_j=0 ,\; (j=1,...,q)$$

 

Suy ra $p+q$ cột $\begin{pmatrix} m_1 \\ 0 \end{pmatrix},...,\begin{pmatrix} m_p \\ 0 \end{pmatrix},\begin{pmatrix} t_1 \\ n_1 \end{pmatrix},...,\begin{pmatrix} t_q \\ n_q \end{pmatrix}$ độc lập tuyến tính, do đó

$$rank \begin{pmatrix}M & T\\ O& N\end{pmatrix} \ge p+q=rank M+rank N$$

 

Bằng cách thay đổi vị trí các cột của $\begin{pmatrix}M & T\\ O& N\end{pmatrix}$ (không làm thay đổi hạng), ta cũng có

 

$$rank \begin{pmatrix}T & M\\ N& O\end{pmatrix} \ge rank M+rank N$$

 

Áp dụng:

 

$$rank \begin{pmatrix}A &B \\0 &D-CA^{-1}B \end{pmatrix} \ge rank A+rank (D-CA^{-1}B)$$

$$\Rightarrow rank (D-CA^{-1}B)=0 \Leftrightarrow D=CA^{-1}B$$

$$\Rightarrow |AD|=|BC|$$

 

Nếu $|A|=0$.

 

* Nếu $|B|=|C|=0$ thì hiển nhiên $|AD|=|BC|=0$

 

* Nếu,$|B| \neq 0$ hoặc $|C|\neq 0$, chẳng hạn $|B| \neq 0$, do đó $B^{-1}$ tồn tại.

 

$$\begin{pmatrix}I &0 \\-DB^{-1} &I \end{pmatrix} \begin{pmatrix}A &B \\C &D \end{pmatrix}=\begin{pmatrix}A &B \\C-DB^{-1}A &0 \end{pmatrix}$$

 

$$\Rightarrow rank \begin{pmatrix}A &B \\C-DB^{-1}A &0 \end{pmatrix}=rank \begin{pmatrix}A &B \\C &D \end{pmatrix}=n$$

 

Lại có: $$rank \begin{pmatrix}A &B \\C-DB^{-1}A &0 \end{pmatrix} \ge rank B+rank(C-DB^{-1}A)$$

$$\Rightarrow (C-DB^{-1}A)=0 \Leftrightarrow C=DB^{-1}A$$

 

$$\Rightarrow |BC|=|AD|$$

 

Vậy $\begin{vmatrix} \left |A \right | & \left |B \right |\\ \left |C \right | & \left |D \right | \end{vmatrix}=|AD|-|BC|=0$




#540911 Chứng minh A chéo hoá được

Đã gửi bởi phudinhgioihan on 15-01-2015 - 15:13 trong Đại số tuyến tính, Hình học giải tích

cài này ở đâu ra?

 

Mình đã ghi rõ trên kia, xem chứng minh tổng quát ở:

"Chứng minh: Xem trang 67, Đại số 2, Jean Marie Monier.

 

File pdf: http://diendantoanho...n-marie-monier/"

 

Với $P(x)=a\prod_{i=1}^n(x-x_i)$ , $Q_k(x)=\prod_{1 \le i \le n, \; i \neq k}(x-x_i)$

Đa thức $\sum_{k=1}^n \dfrac{Q_k(x)}{Q_k(x_k)}-1$ bậc không quá $n-1$ nhưng lại có $n$ nghiệm do đó là đa thức $0$ hay

$\sum_{k=1}^n \dfrac{Q_k(x)}{Q_k(x_k)}=1$




#540868 Tìm giới hạn $\lim \frac{n^a}{a^n} (a>...

Đã gửi bởi phudinhgioihan on 14-01-2015 - 23:14 trong Giải tích

Tìm giới hạn $\lim \frac{n^a}{a^n} (a>1)$

 

Kiên :ukliam2:

 

Do $a>1$ nên $a^\frac{1}{a}=1+h \;, h>0$

 

Với $n \ge 2$, $(1+h)^n=\sum_{k=0}^n C_n^kh^k \ge \sum_{k=0}^2 C_n^k h^k=1+nh+\frac{1}{2}n(n-1)h^2$

 

Với $n \ge 2 $

 

Ta có: $0<\dfrac{n^a}{a^n}=\left( \dfrac{n}{a^\frac{n}{a}} \right)^a=\left( \dfrac{n}{(1+h)^n} \right)^a \le \left(\dfrac{n}{1+nh+\frac{1}{2}n(n-1)h^2} \right)^a \underset{n \to +\infty}{\longrightarrow} 0$

 

Vậy $\lim_{n \to +\infty} \dfrac{n^a}{a^n}=0$




#540862 Ma trận lũy linh

Đã gửi bởi phudinhgioihan on 14-01-2015 - 22:43 trong Đại số tuyến tính, Hình học giải tích

Anh có thể nêu các bước chứng minh được không ạ

 

Xin chứng minh cho trường hợp thường sử dụng (tổng quát tương tự):
Ma trận $A$ cấp $n \in \mathbb{N}^*$ thực hoặc phức đều đồng dạng với ma trận tam giác (tam giác hóa được) trong $\mathbb{C}$.

 

Do ma trận tam giác trên đồng dạng với ma trận tam giác dưới cho nên ta chỉ cần chứng minh mọi ma trận phức đều đồng dạng với một ma trận tam giác trên.

Ký hiệu $\chi_A$ là đa thức đặc trưng của $A$, gtr:giá trị riêng, vtr: vecto riêng, $T_{n}(\mathbb{C})$ là tập các ma trận tam giác trên có các phần tử thuộc $\mathbb{C}$.

 

Quy nạp theo n.

Tính chất là tầm thường với n=1.

Giả sử tính chất trên đúng với $n\in \mathbb{N}^*$ và giả sử $A \in \mathbf{M}_{n+1}(\mathbb{C})$. Khi đó,$A$ có ít nhất một gtr $\lambda_1 \in \mathbb{C}$ và một vtr liên kết $V_1 \in \mathbf{M}_{n+1}(\mathbb{C})$, do đó tồn tại $B\in\mathbf{M}_{1,n}(\mathbb{C}), A_1 \in \mathbf{M}_{n}(\mathbb{C})$ sao cho $A \sim \begin{pmatrix}\lambda_1 & B \\ 0 & A_1 \end{pmatrix}$.

 

Ta có $\forall \lambda \in \mathbb{C},\; \chi_A(\lambda)=\det \begin{pmatrix}\lambda_1-\lambda & B \\ 0 & A_1-\lambda I_n \end{pmatrix}=(\lambda_1-\lambda) \chi_{A_1}(\lambda)$

 

Theo giả thiết quy nạp, tồn tại $C \in \mathbf{GL}_n(\mathbb{C})$ và $D \in T_{n}(\mathbb{C})$ sao cho $A_1=CDC^{-1}$

 

Đặt $E=\begin{pmatrix}1 & 0 \\ 0 & C \end{pmatrix} \in \mathbf{M}_{n+1}(\mathbb{C})$, khả nghịch và $E^{-1}=\begin{pmatrix}1 & 0 \\ 0 & C^{-1} \end{pmatrix}$

 

Đặt $F=\begin{pmatrix}\lambda_1 & X \\ 0 & D \end{pmatrix} \in T_{n+1}(\mathbb{C})$ với $X \in \mathbf{M}_{1,n}(\mathbb{C})$ sẽ xác định sau để $\begin{pmatrix}\lambda_1 & B \\ 0 & A_1 \end{pmatrix}=E FE^{-1}$

 

Ta có: $EFE^{-1}=\begin{pmatrix}\lambda_1 & XC^{-1} \\ 0 & A_1 \end{pmatrix}$

 

Chọn $X=BC$, khi đó ta được $\begin{pmatrix}\lambda_1 & B \\ 0 & A_1 \end{pmatrix}=E FE^{-1}$

 

Điều này chứng tỏ $A \sim F$

 

Vậy $A$ tam giác hóa được trong $\mathbb{C}$




#540824 Ma trận lũy linh

Đã gửi bởi phudinhgioihan on 14-01-2015 - 20:13 trong Đại số tuyến tính, Hình học giải tích

Mình cũng chưa từng nghe đến định lý này. Nếu như điều này xảy ra thì khá vô lý vì hoá ra mọi ma trận thực đều có giá trị riêng thực, nếu thế thì mọi đa thức thực đều có nghiệm thực? Phải chăng bạn funcalys đang nhầm đưa được về ma trận tam giác trên bằng khử Gauss với đồng dạng.

 

Chính xác phải là mọi ma trận đều đồng dạng với một ma trận tam giác (tam giác hóa được) trong một trường số mở rộng nào đó.

Trường hợp hay dùng tới : Mọi ma trận thực hoặc phức đều tam giác hóa được trong $\mathbb{C}$




#540821 Tìm ma trận giao hoán

Đã gửi bởi phudinhgioihan on 14-01-2015 - 19:51 trong Đại số tuyến tính, Hình học giải tích

Anh ơi chỗ này đâu suy ra được ạ @@

 

Chỗ đó anh làm tắt, $B$ có 2 cột đầu toàn 0 rồi, thay $b_3$ vào thì ra liền $b_3=0$, tính toán đơn giản nên khỏi ghi vậy.